Những câu hỏi liên quan
Kuvip Yb
Xem chi tiết
Akai Haruma
17 tháng 6 2021 lúc 22:55

Lời giải:

Gọi $d$ là ƯCLN của $a$ và $b$. Khi đó $a=dx, b=dy$ với $x,y$ nguyên dương và nguyên tố cùng nhau

Ta có:

$d=15$

BCNN$(a,b)=dxy=2835$

$\Rightarrow xy=189$

Mà $x,y$ là 2 số nguyên dương nguyên tố cùng nhau nên $(x,y)=(1,189), (189,1), (27,7), (7,27)$

$\Rightarrow (a,b)=(15,2835), (2835, 15), (405,105), (105,405)$

Bình luận (0)
I lay my love on you
Xem chi tiết
Nguyễn Tất Đạt
13 tháng 1 2019 lúc 21:47

Bài 2: Giả sử tồn tại x,y nguyên dương t/m đề, khi đó pt cho tương đương:

\(4x^2+4y^2-12x-12y=0\Leftrightarrow\left(2x+3\right)^2+\left(2y+3\right)^2=18\)

Ta thấy: \(18=9+9=3^2+3^2\). Mà x,y thuộc Z+ nên \(\hept{\begin{cases}2x+3=3\\2y+3=3\end{cases}\Leftrightarrow}\hept{\begin{cases}x=0\\y=0\end{cases}}\)

Vậy cặp nghiệm nguyên t/m pt là (x;y) = (0;0)

Bình luận (0)
Nguyễn Tất Đạt
13 tháng 1 2019 lúc 22:29

Làm lại bài 2 :v (P/S: Bạn bỏ bài kia đi nhé)

\(4x^2+4y^2-12x-12y=0\Leftrightarrow\left(2x-3\right)^2+\left(2y-3\right)^2=18\)

Ta thấy: \(18=9+9=3^2+3^2\). Mà x,y thuộc Z+ nên \(\hept{\begin{cases}2x-3=3\\2y-3=3\end{cases}\Leftrightarrow}\hept{\begin{cases}x=3\\y=3\end{cases}}\)

Vậy (x;y) = (3;3)

Bình luận (0)
Tiến Nguyễn Minh
Xem chi tiết
✰๖ۣۜŠɦαɗøω✰
27 tháng 3 2020 lúc 8:31

Bài 1 : 

Phương trình <=> 2x . x2 = ( 3y + 1 ) + 15

Vì \(\hept{\begin{cases}3y+1\equiv1\left(mod3\right)\\15\equiv0\left(mod3\right)\end{cases}\Rightarrow\left(3y+1\right)^2+15\equiv1\left(mod3\right)}\)

\(\Rightarrow2^x.x^2\equiv1\left(mod3\right)\Rightarrow x^2\equiv1\left(mod3\right)\)

( Vì số  chính phương chia 3 dư 0 hoặc 1 ) 

\(\Rightarrow2^x\equiv1\left(mod3\right)\Rightarrow x\equiv2k\left(k\inℕ\right)\)

Vậy \(2^{2k}.\left(2k\right)^2-\left(3y+1\right)^2=15\Leftrightarrow\left(2^k.2.k-3y-1\right).\left(2^k.2k+3y+1\right)=15\)

Vì y ,k \(\inℕ\)nên 2k . 2k + 3y + 1 > 2k .2k - 3y-1>0

Vậy ta có các trường hợp: 

\(+\hept{\begin{cases}2k.2k-3y-1=1\\2k.2k+3y+1=15\end{cases}\Leftrightarrow\hept{\begin{cases}2k.2k=8\\3y+1=7\end{cases}\Rightarrow}k\notinℕ\left(L\right)}\)

\(+,\hept{\begin{cases}2k.2k-3y-1=3\\2k.2k+3y+1=5\end{cases}\Leftrightarrow\hept{\begin{cases}2k.2k=4\\3y+1=1\end{cases}\Rightarrow}\hept{\begin{cases}k=1\\y=0\end{cases}\left(TM\right)}}\)

Vậy ( x ; y ) =( 2 ; 0 ) 

Bình luận (0)
 Khách vãng lai đã xóa
Phùng Gia Bảo
27 tháng 3 2020 lúc 9:14

Bài 3: 

Giả sử \(5^p-2^p=a^m\)    \(\left(a;m\inℕ,a,m\ge2\right)\)

Với \(p=2\Rightarrow a^m=21\left(l\right)\)

Với \(p=3\Rightarrow a^m=117\left(l\right)\)

Với \(p>3\)nên p lẻ, ta có

\(5^p-2^p=3\left(5^{p-1}+2.5^{p-2}+...+2^{p-1}\right)\Rightarrow5^p-2^p=3^k\left(1\right)\)    \(\left(k\inℕ,k\ge2\right)\)

Mà \(5\equiv2\left(mod3\right)\Rightarrow5^x.2^{p-1-x}\equiv2^{p-1}\left(mod3\right),x=\overline{1,p-1}\)

\(\Rightarrow5^{p-1}+2.5^{p-2}+...+2^{p-1}\equiv p.2^{p-1}\left(mod3\right)\)

Vì p và \(2^{p-1}\)không chia hết cho 3 nên \(5^{p-1}+2.5^{p-2}+...+2^{p-1}⋮̸3\)

Do đó: \(5^p-2^p\ne3^k\), mâu thuẫn với (1). Suy ra giả sử là điều vô lý

\(\rightarrowĐPCM\)

Bình luận (0)
 Khách vãng lai đã xóa
Lê Nhật Khôi
27 tháng 3 2020 lúc 10:53

Bài 4:

Ta đặt: \(S=6^m+2^n+2\)

TH1: n chẵn thì:

\(S=6^m+2^n+2=6^m+2\left(2^{n-1}+1\right)\)

Mà \(2^{n-1}+1⋮3\Rightarrow2\left(2^{n-1}+1\right)⋮6\Rightarrow S⋮6\)

Đồng thời S là scp

Cho nên: \(S=6^m+2\left(2^{n-1}\right)=\left(6k\right)^2\)

\(\Leftrightarrow6^m+6\left(2^{n-2}-2^{n-3}+...+2-1\right)=36k^2\)

Đặt: \(A\left(n\right)=2^{n-2}-2^{n-3}+...+2-1=2^{n-3}+...+1\)là số lẻ

Tiếp tục tương đương: \(6^{m-1}+A\left(n\right)=6k^2\)

Vì A(n) lẻ và 6k^2 là chẵn nên: \(6^{m-1}\)lẻ\(\Rightarrow m=1\)

Thế vào ban đầu: \(S=8+2^n=36k^2\)

Vì n=2x(do n chẵn) nên tiếp tục tương đương: \(8+\left(2^x\right)^2=36k^2\)

\(\Leftrightarrow8=\left(6k-2^x\right)\left(6k+2^x\right)\)

\(\Leftrightarrow2=\left(3k-2^{x-1}\right)\left(3k+2^{x-1}\right)\)

Vì \(3k+2^{x-1}>3k-2^{x-1}>0\)(lớn hơn 0 vì 2>0 và \(3k+2^{x-1}>0\))

Nên: \(\hept{\begin{cases}3k+2^{x-1}=2\\3k-2^{x-1}=1\end{cases}}\Leftrightarrow6k=3\Rightarrow k\notin Z\)(loại)

TH2: n là số lẻ

\(S=6^m+2^n+2=\left(2k\right)^2\)(do S chia hết cho 2 và S là scp)

\(\Leftrightarrow3\cdot6^{m-1}+2^{n-1}+1=2k^2\)là số chẵn

\(\Rightarrow3\cdot6^{m-1}+2^{n-1}\)là số lẻ

Chia tiếp thành 2TH nhỏ: 

TH2/1: \(3\cdot6^{m-1}\)lẻ và \(2^{n-1}\)chẵn với n là số lẻ

Ta thu đc: m=1 và thế vào ban đầu

\(S=2^n+8=\left(2k\right)^2\)(n lớn hơn hoặc bằng 3)

\(\Leftrightarrow2^{n-2}+2=k^2\)

Vì \(k^2⋮2\Rightarrow k⋮2\Rightarrow k^2=\left(2t\right)^2\)

Tiếp tục tương đương: \(2^{n-2}+2=4t^2\)

\(\Leftrightarrow2^{n-3}+1=2t^2\)

\(\Leftrightarrow2^{n-3}\)là số lẻ nên n=3

Vậy ta nhận đc: \(\left(m;n\right)=\left(1;3\right)\)

TH2/2: \(3\cdot6^{m-1}\)là số chẵn và \(2^{n-1}\)là số lẻ

Suy ra: n=1

Thế vào trên: \(6^m+4=4k^2\)

\(\Leftrightarrow6^m=\left(2k-2\right)\left(2k+2\right)\)

\(\Leftrightarrow\hept{\begin{cases}2k-2=6^q\\2k+2=6^p\end{cases}}\Rightarrow p+q=m\)

Và \(6^p-6^q=4\)

\(\Leftrightarrow6^q\left(6^{p-q}-1\right)=4\Leftrightarrow6^q\le4\Rightarrow q=1\)(do là tích 2 stn)

\(\Rightarrow k\notin Z\)

Vậy \(\left(m;n\right)=\left(1;3\right)\)

P/S: mk không kiểm lại nên có thể sai

Bình luận (0)
 Khách vãng lai đã xóa
Mai Tiến Đỗ
Xem chi tiết
Tran Nguyen
Xem chi tiết
Nguyễn Anh Quân
13 tháng 1 2018 lúc 19:58

Bài 13 :

Có : c = (a-b).[-(a-b)] = -(a-b)^2

Vì a khác b => a-b khác 0 => (a-b)^2 > 0

=> c = -(a-b)^2 < 0

=> c là số âm

Tk mk nha

Bình luận (0)
Tran Nguyen
13 tháng 1 2018 lúc 20:00

làm dùm mình bài 14 luôn nha bạn

Bình luận (0)
Nguyễn Nhã Thanh
13 tháng 1 2018 lúc 20:04

bài 13

c là số âm

bài 14

a,a.b=-6

a/b=1,b/a=6

a/b=2,b/a=3

còn cả p/s nữa

Bình luận (0)
Kill Myself
Xem chi tiết
Công Tử Họ Nguyễn
9 tháng 10 2018 lúc 21:27

Giả sử (x;y) là cặp số nguyên dương cần tìm. Khi đó ta có: 
(xy-1) I (x^3+x) => (xy-1) I x.(x^2+1) (1) 
Do (x; xy-1) =1 ( Thật vậy: gọi (x;xy-1) =d => d I x => d I xy => d I 1). 
Nên từ (1) ta có: 
(xy-1) I (x^2+1) 
=> (xy-1) I (x^2+1+xy -1) => (xy-1) I (x^2+xy) => (xy-1) I x.(x+y) => (xy-1) I (x+y) 
Điều đó có nghĩa là tồn tại z ∈ N* sao cho: 
x+y = z(xy-1) <=> x+y+z =xyz (2) 

[Đây lại có vẻ là 1 bài toán khác] 
Do vai trò bình đẳng nên ta giả sử: x ≥ y ≥ z. 
Từ (2) ta có: x+y+z ≤ 3x => 3x ≥ xyz => 3 ≥ yz ≥ z^2 => z=1 
=> 3 ≥ y => y ∈ {1;2;3} 
Nếu y=1: x+2 =x (loại) 
Nếu y=2: (2) trở thành x+3 =2x => x=3 
Nếu y=3: x+4 = 3x => x=2 (loại vì ta có x≥y) 
Vậy khi x ≥ y ≥ z thì (2) có 1 nghiệm (x;y;z) là (3;2;1) 
Hoán vị vòng quanh được 6 nghiệm là: .....[bạn tự viết nhé] 

Vậy bài toán đã cho có 6 nghiệm (x;y) là : .... [viết y chang nhưng bỏ z đi]

Bình luận (0)
Kim
9 tháng 10 2018 lúc 21:27

 Giả sử (x;y) là cặp số nguyên dương cần tìm. Khi đó ta có: 
(xy-1) I (x^3+x) => (xy-1) I x.(x^2+1) (1) 
Do (x; xy-1) =1 ( Thật vậy: gọi (x;xy-1) =d => d I x => d I xy => d I 1).
Nên từ (1) ta có: 
(xy-1) I (x^2+1) 
=> (xy-1) I (x^2+1+xy -1) => (xy-1) I (x^2+xy) => (xy-1) I x.(x+y) => (xy-1) I (x+y) 
Điều đó có nghĩa là tồn tại z ∈ N* sao cho: 
x+y = z(xy-1) <=> x+y+z =xyz (2) 

[Đây lại có vẻ là 1 bài toán khác] 
Do vai trò bình đẳng nên ta giả sử: x ≥ y ≥ z. 
Từ (2) ta có: x+y+z ≤ 3x => 3x ≥ xyz => 3 ≥ yz ≥ z^2 => z=1 
=> 3 ≥ y => y ∈ {1;2;3} 
Nếu y=1: x+2 =x (loại) 
Nếu y=2: (2) trở thành x+3 =2x => x=3 
Nếu y=3: x+4 = 3x => x=2 (loại vì ta có x≥y) 
Vậy khi x ≥ y ≥ z thì (2) có 1 nghiệm (x;y;z) là (3;2;1) 
Hoán vị vòng quanh được 6 nghiệm là: .....[bạn tự viết nhé] 

Vậy bài toán đã cho có 6 nghiệm (x;y) là : .... [viết y chang nhưng bỏ z đi]

Bình luận (0)
Hoàng Thế Hải
9 tháng 10 2018 lúc 21:34

Xét x= 1 => \(\dfrac{2}{y-1}\in\mathbb N\), từ đó có \(y=2\vee y=3\)

Xét y=1 => \(\dfrac{x^3+x}{x-1}=x^2+x+2+\dfrac{2}{x-1}\in\mathbb N\), từ đó có \(x=2\vee x=3\)

Xét \(x\ge 2\) hoặc \(y\ge 2\) . Ta có : \((x,xy-1)=1\). Do đó :

\(xy-1|x^3+x\Rightarrow xy-1|x^2+1\Rightarrow xy-1|x+y\)

=> \(x+y\ge xy-1\Rightarrow (x-1)(y-1)\le 2\). Từ đó có \((x-1)(y-1)=1\ \vee (x-1)(y-1)=2\) 

=> x = y = 2 ( loại ) hoặc x = 2 ; y = 3 hoặc x = 3 ; y= 2

Vậy các cặp số ( x;y ) thỏa mãn là (1;2),(2;1),(1;3),(3;1),(2;3),(3;2)

Bình luận (0)
I lay my love on you
Xem chi tiết
Đỗ Bảo Châu
7 tháng 10 2021 lúc 19:50

Mình không biết nha tạm thời bạn hỏi bạn khác đi 😅

Bình luận (0)
 Khách vãng lai đã xóa
Bùi Minh Quân
Xem chi tiết
Phạm Minh
Xem chi tiết
Phạm Minh
16 tháng 6 2020 lúc 20:40

Ai giúp em với ạ

Bình luận (0)
 Khách vãng lai đã xóa
Nguyễn Linh Chi
16 tháng 6 2020 lúc 21:06

1. Ta có: \(x^2-2xy-x+y+3=0\)

<=> \(x^2-2xy-2.x.\frac{1}{2}+2.y.\frac{1}{2}+\frac{1}{4}+y^2-y^2-\frac{1}{4}+3=0\)

<=> \(\left(x-y-\frac{1}{2}\right)^2-y^2=-\frac{11}{4}\)

<=> \(\left(x-2y-\frac{1}{2}\right)\left(x-\frac{1}{2}\right)=-\frac{11}{4}\)

<=> \(\left(2x-4y-1\right)\left(2x-1\right)=-11\)

Th1: \(\hept{\begin{cases}2x-4y-1=11\\2x-1=-1\end{cases}}\Leftrightarrow\hept{\begin{cases}x=0\\y=-3\end{cases}}\)

Th2: \(\hept{\begin{cases}2x-4y-1=-11\\2x-1=1\end{cases}}\Leftrightarrow\hept{\begin{cases}x=1\\y=3\end{cases}}\)

Th3: \(\hept{\begin{cases}2x-4y-1=1\\2x-1=-11\end{cases}}\Leftrightarrow\hept{\begin{cases}x=-5\\y=-3\end{cases}}\)

Th4: \(\hept{\begin{cases}2x-4y-1=-1\\2x-1=11\end{cases}}\Leftrightarrow\hept{\begin{cases}x=6\\y=3\end{cases}}\)

Kết luận:...

Bình luận (0)
 Khách vãng lai đã xóa
Nguyễn Linh Chi
16 tháng 6 2020 lúc 21:12

2. \(y^2+1\ge1>0;2x^2+x+1>0\) với mọi x; y 

=> x + 5 > 0 

=>  \(y^2+1=\frac{x+5}{2x^2+x+1}\ge1\)

<=> \(x+5\ge2x^2+x+1\)

<=> \(x^2\le2\)

Vì x nguyên => x = 0 ; x = 1; x = -1 

Với x = 0 ta có: \(y^2+1=5\Leftrightarrow y=\pm2\)

Với x = 1 ta có: \(y^2+1=\frac{3}{2}\)loại vì y nguyên 

Với x = -1 ta có: \(y^2+1=2\Leftrightarrow y=\pm1\)

Vậy Phương trình có 4 nghiệm:...

Bình luận (0)
 Khách vãng lai đã xóa